- PowerScore Staff
- Posts: 5948
- Joined: Mar 25, 2011
- Wed Jun 27, 2018 12:37 pm
#47078
Complete Question Explanation
(The complete setup for this game can be found here: lsat/viewtopic.php?t=8685)
The correct answer choice is (B)
Answer choice (A) is incorrect because R sits in seat G2.
Answer choice (B) is the correct answer.
Answer choice (C) is incorrect because this answer forms a QS or SQ block, and thus neither Q nor S could sit in the same row as R, a violation of the fourth rule.
Answer choices (D) and (E) are incorrect because R sits in row G, whereas T and U sit in row H.
(The complete setup for this game can be found here: lsat/viewtopic.php?t=8685)
The correct answer choice is (B)
Answer choice (A) is incorrect because R sits in seat G2.
Answer choice (B) is the correct answer.
Answer choice (C) is incorrect because this answer forms a QS or SQ block, and thus neither Q nor S could sit in the same row as R, a violation of the fourth rule.
Answer choices (D) and (E) are incorrect because R sits in row G, whereas T and U sit in row H.
Dave Killoran
PowerScore Test Preparation
Follow me on X/Twitter at http://twitter.com/DaveKilloran
My LSAT Articles: http://blog.powerscore.com/lsat/author/dave-killoran
PowerScore Podcast: http://www.powerscore.com/lsat/podcast/
PowerScore Test Preparation
Follow me on X/Twitter at http://twitter.com/DaveKilloran
My LSAT Articles: http://blog.powerscore.com/lsat/author/dave-killoran
PowerScore Podcast: http://www.powerscore.com/lsat/podcast/